• Shuffle
    Toggle On
    Toggle Off
  • Alphabetize
    Toggle On
    Toggle Off
  • Front First
    Toggle On
    Toggle Off
  • Both Sides
    Toggle On
    Toggle Off
  • Read
    Toggle On
    Toggle Off
Reading...
Front

Card Range To Study

through

image

Play button

image

Play button

image

Progress

1/46

Click to flip

Use LEFT and RIGHT arrow keys to navigate between flashcards;

Use UP and DOWN arrow keys to flip the card;

H to show hint;

A reads text to speech;

46 Cards in this Set

  • Front
  • Back
1. 24 year old comatose pt from a car accident with eye movements that are non-purposeful.
ANS: The patient is in a neurovegetative state and although his reticular activity is in tact, connections are compromised.
1. Which is NOT a measurement of neurological function
ANS: IQ
1. A previously healthy 38 year old male is unresponsive. His BP is 210/90. He attends parties where there is cocaine. Which would you NOT do in assessing this patient?
ANS: a lumbar puncture to r/o subdural bleed (subarachnoid doesn’t communicate with subdural bleed)
1. 68 year old male with hx of using tobacco has shortness of breath over 6 mo. Can’t do manual tasks. Suspect COPD- O2 stats @ room air is 80%. All are true EXCEPT:
ANS: His hypoxia is likely caused by a true pulmonary shunt
1. 78 year old male with CHF. Heart displaced to the left. Vocal and tactile frematis on the right with decreased breath sounds.
ANS: right pleural effusion
1. An example of hypoxia due to a pure Right to Left shunt is:
ANS: A 4 year old with a VSD
1. Increased leukocytes are NOT found in
ANS: Migraine
T or F: Lymphocytes are increased in a left shift
False: 1. ANS: Lymphocytes are NOT increased in a Left shift
1. 72 year old female with metastatic breast cancer presents with weakness. HR is 110, BP 88/45. Mucous membranes are dry. Na is 133, BUN is 28, and normal creatinine of 1.8. All are true EXCEPT:
ANS: patient is dehydrated and would benefit from administration of free H2O.
1. Which is least likely to increase eosinophils
ANS: pneumococcal pneumonia (remember, think NAACP)
1. All of the following can increase body temperature. Which is NOT an exogenous pyrogen?
ANS: cytokines
1. Which of the following statements is TRUE regarding hyperthermia?
ANS: Hyperthermia is an increase in temperature with a NORMAL hypothalamic set point
1. Which of the following scenarios is mechanical lowering of body temperature NOT indicated?
ANS: a case of west nile virus in an elderly male with a temp of 38 degrees C.
1. A 69 year old male with 2 week history of retrosternal chest pain, increased dyspnea and dizziness after exercise. BP is 125/95 (narrow pulse pressure) and pulse of 76. Patient is NOT a smoker. On auscultaion of the heart he is noted to have a harsh, late peaking, crescendo-decrescendo systolic ejection murmur. What is his likely diagnosis?
ANS: critical aortic stenosis
1. What is the mechanism of aortic stenosis?
ANS: supply/demand mismatch with sub endocardial ischemia secondary to increased left ventricular pressure in face of significant thickening of the left ventricular myocardium.
1. A 48 year old female with Graves disease has pedal edema, orthopnea, dyspnea, and a myxomatous mitral valve. She has a loud holosystolic murmur that radiates to the left axillae. What is the best course of action for her?
ANS: she should undergo a valve replacement with a MECHANICAL valve (because she is young) and be on anticoagulation for life
1. All are true of aortic insufficiency EXCEPT:
ANS: murmur of aortic insufficiency is loud, low pitch, and best heard at the beginning of systole over the Left precordium (false b/c AI is in DIASTOLE)
1. A 42 year old female plays tennis. She has jaw pain, mild nausea, and shortness of breat. Crushing substernal chest pain. She smokes. Her glucose is 125. EKG shows ST elevation, ST impressions, Reciprocal changes across the anterolateral wall of heart. She most likely has:
ANS: Rupture of a vulnerable plaque in a major coronary with subsequent formation of an occlusive thrombus eliminating anterograde blood flow to a portion of the myocardium.
1. ANS: Beta blockers
1. A 55 year old male with history of diabetes, smoking, etc… which risk factor affects him the most?
ANS: cigarette smoking
1. Which scenario is NOT an absolute indication for CAB?
ANS: severe (>90%) stenosis in mid portion of the LAD with patent collateral circulation.
1. A young Japanese medical student presents with crushing substernal chest pain. His condition:
ANS: often requires stenting and or thrombolytic therapy
1. A 59 year old post menopausal woman who is overweight and has diabetes. Her triglycerides are 400. Which is NOT immediately recommended?
ANS: pharmacotherapy to decrease patient’s weight
1. What influences what and EKG does?
1. To determine heart rate from EKG (300/# of big boxes).
1. Which interval in the EKG represents the delay between SA and AV node in the heart?
ANS: the PR interval
1. Which of the following is correct about Deep Venous Throbosis and PE?
ANS: A normal V/Q scan excludes the diagnosis of PE with 75 % accuracy
1. A patient with orthopnea, dyspnea and an enlarged heart:
ANS: he is likely to have adaptive mechanisms for his heart failure which would include ventricular remodeling and neurohormone activity.
If a patient has a HR of 150
1. (It is a 2:1 atrial flutter until proven otherwise!)
1. With systolic heart failure, which of the following is NOT true?
ANS: plasma levels of NE will be decreased (false because everything neuroendocrine will be increased)
1. Treatment for dilated cardiomyopathy includes:
ANS: DECREASE in Na intake
1. A 23 year old blacked out:
ANS: ECG is diagnostic test of choice and should be done ASAP
1. Which is NOT a dilated cardiomyopathy?
ANS: Amyloid cardiomyopathy
1. Dimentia of Alzheimers disease (DAT) is best correlated with neuropathologial findings. Severity depends on:
ANS: degree of neuronal synaptic loss
1. An 86 year old male develops flu like symptoms followed by a bacterial lower pneumonia. His Serum Na is 115 and Urine Na is 35. Serum Os is 240 and Urine Os is 600. He has:
ANS: SIADH (if urine Os is greater than serum, then you are inappropriately secreting ADH)
1. Which of the following is NOT a cardinal sign of Parkinsons:
ANS: Flacid paresis
1. A 60 year old male with advanced colon cancer presents with 4+ Edema, large protein in urine w/o casts. (Nephrotic syndrome picture). All are true EXCEPT:
ANS: the presence of RBC casts or dysmorphic RBCs in the urine are commonly found in Nephrotic syndrome. (this is false b/c this is descriptive of NEPHRITIC syndrome)
1. Emphysema is characterized by:
ANS: FEV1/FVC decreased
1. All are examples of acute renal failure EXCEPT:
ANS: Ureteral obstruction
1. Not likely to occur in dehydration and volume depletion:
ANS: an increase in UNa via aldosterone stimulation
1. 38 year old male is tachypnic with a BP of 105/64. May have O.D. He has low serum Na, a low pH, PCO2… In determining the cause of his hyponatremia the 1st step is to:
ANS: compare a calculated to a measured serum osmolality
1. From the patient in the previous question… how can you tell he is volume depleted?
ANS: by the BUN and Creatnine ratio
1. A seven year old child with cocacola colored urine and periorbital edema. He has low complement. He is most likely to have:
ANS: Acute post infectious glomerulonephritis.
1. The best way to distinguish chronic from acute renal failure?
ANS: In chronic renal failure, small echogenic kidneys on ultrasound
1. What is the best description of epilepsy?
ANS: Recurrent, unprovoked seizures of unexplained or unknown etiology.
1. A compensatory response to metabolic alkylosis is characterized by:
ANS: increased arterial blood pCO2
1. A 67 year old male with history of smoking and progressive shortness of breath. Arterial blood gases: 7.32/58/65/33 (pH/pCO2/PO2/HCO3). What is the primary acid-base disorder?